LSAT and Law School Admissions Forum

Get expert LSAT preparation and law school admissions advice from PowerScore Test Preparation.

 emigini
  • Posts: 6
  • Joined: Aug 13, 2014
|
#16096
Hello,

I struggled between answers A and E for this question. I ended up choosing A (incorrectly) because of the journalist's claim that "it is not fair for those who have worked 40 or more years to deprive others of opportunities."
I understand why E is a good answer, but I don't understand why A is not. Could you explain?

Thank you!
 Steve Stein
PowerScore Staff
  • PowerScore Staff
  • Posts: 1153
  • Joined: Apr 11, 2011
|
#16119
Hi emigini,

That's a good question. The author argues for mandatory retirement based in part on the notion that it is not fair for those who have worked for 40+ years to deprive others of employment.

The right answer to his assumption question will be the one that the author's argument depends on--so if we take the right answer choice away, that will hurt the author's argument.

Answer choice (A) provides the assumption that anyone who has worked for 40 years is at least 65. What if we take this assumption away (or 'negate' it)? Then some people who have worked for 40 years are under 65...does that hurt the author's argument about mandatory retirement? Not at all--it may suggest that mandatory retirement should come even earlier.

I hope that's helpful! Please let me know whether this is clear--thanks!

~Steve
 emigini
  • Posts: 6
  • Joined: Aug 13, 2014
|
#16127
That does make sense. Thank you!
 Steve Stein
PowerScore Staff
  • PowerScore Staff
  • Posts: 1153
  • Joined: Apr 11, 2011
|
#16133
Hi.

Thanks! I really appreciate your quick response, and I'm glad that you found that helpful!

~Steve
 lsatjourneygirl
  • Posts: 22
  • Joined: May 03, 2016
|
#25205
How would you get the logical negation of answer choices A, B, C?

Aren't there different degrees as to how far you can go with a negation of say answer choice?

For instance, couldn't you just change "anyone" to "none"

or change "anyone" to "none" and change "at least" to "less than"?

thank you! I hope this makes sense!
 Nikki Siclunov
PowerScore Staff
  • PowerScore Staff
  • Posts: 1362
  • Joined: Aug 02, 2011
|
#25392
Hi,

The logical negation of answer choices (A), (B), and (C) is as follows:
(A) Not everyone who has worked 40 years is at least 65 years old (in other words, at least some people who have worked for 40 years are younger than 65).
Whether or not this is true has no bearing on the conclusion of the argument.
(B)Not all young people entering the job market are highly trained professionals (in other words, some young people entering the job market are not highly trained professionals.
So what? The logical opposite of this answer choice has no effect on the conclusion.
(C) It is not unfair for a person not to get a job in the profession for which that person was trained. In other words, people aren't entitled to a job in the profession for which they have trained.
As you can see, if it important too look at these statements holistically and negate their core meaning. It is even more important to simplify the meaning of the negated statement so as not to deal with double- or triple-negatives.

For more information on Complex Statement Negation, please refer to this Blog Post:

Negating conditional statements on the LSAT

Negating Compound and Conditional Statements

Let me know if this helps.

Thanks!
 brcibake
  • Posts: 55
  • Joined: Jul 19, 2017
|
#39262
I thought the answer was C. My reasoning was because the stimulus seemed to steer in the direction of how unfair it is for the younger population to be disadvantaged in such a way. Could you explain the correct answer's (E) reasoning and why C is wrong?
Thank you
 Adam Tyson
PowerScore Staff
  • PowerScore Staff
  • Posts: 5153
  • Joined: Apr 14, 2011
|
#39478
Thanks for the question, brcibake. The stimulus tells us that there are two "unacceptable" outcomes of ending mandatory retirement. Not "unfair", but "unacceptable". That's an important distinction! The first unacceptable outcome is young people not getting jobs in their profession. The second is that it's unfair to deprive young people of opportunities.

Answer C combines these two claims. The author never said the first one was unfair, just unacceptable, and he doesn't have to think it's unfair. Maybe he believes it is completely fair but nonetheless harmful to society in some way? The negation of C, making it fair for a person to not get a job in their profession, doesn't harm the argument because it could still be unacceptable for that to happen. Fair does not equal acceptable.

There's also another problem with C: If you happen to interpret "unacceptable" as being synonymous with "unfair", then answer C would not be an assumption of the argument, but a premise, because he said it was unacceptable. Assumptions are always things the author did not say.

I hope that clears it up for you!
 chian9010
  • Posts: 81
  • Joined: Jun 08, 2018
|
#59841
I have a question about E).

I think it is a rephrase of the first sentence in Journalist's argument "many people object to mandatory retirement at age 65 as being arbitrary, arguing that people over 65 make useful contributions."

If the journalist doesn't include the first sentence but directly talks about his own argument saying mandatory retirement should be retained, then I think E is absolutely the journalist's own assumption.

However, in this stimulus, Journalist includes the first sentence which means that many people believe if the retirement ceases to be mandatory at age 65, then many people over 65 will make useful contribution which mean that many people will choose to keep working. Therefore, this is not the journalist's own assumption.

Please help me clarify the confusion.
 Brook Miscoski
PowerScore Staff
  • PowerScore Staff
  • Posts: 418
  • Joined: Sep 13, 2018
|
#62493
Chian,

An assumption doesn't have to be implausible to qualify as an answer. The criteria is that an assumption is essential to the argument but is not explicitly stated. There's nothing in the passage that explicitly states that people will continue working after the age of 65. Also, look at the last sentence of the stimulus. The journalist is discussing a society that already has mandatory retirement at age 65. Thus, the claim about contributions is not a claim about people in the workforce, so it is not implied that people over the age of 65 are already in the workforce.

Get the most out of your LSAT Prep Plus subscription.

Analyze and track your performance with our Testing and Analytics Package.